45 votes
Accepted

A challenging (for me) limit calculation

This limit converges to $\frac{\sqrt3}2$. The idea is that $\sin(x) = x - \frac{x^3}6 + O(x^5)$, so we start with $\frac1{\sqrt n}$ and repeatedly subtract $\frac{x^3}6$. We can approximate this ...
Command Master's user avatar
13 votes
Accepted

Asymptotics of a strange oscillatory function

I will show (unless I made a mistake) $$ f(x) = a\sqrt{x} + O(x^{1/3}), $$ for some constant $a > 0$. For each $x > 0$, let $g(t) = \sin(x/t^2)$. Let $\varepsilon > 0$ to be chosen later ...
Matt Young's user avatar
  • 4,633
12 votes
Accepted

Asymptotics for pairs of positive integers whose harmonic (resp. geometric) mean is an integer

[CORRECTED 8/28] I think the asymptotic ratio is $(4/3)*\ln(2)\approx 0.924196$. $$G(n) = (6/\pi^2)*n*\ln(n) + O(n)$$ $$H(n) = (8\ln(2)/\pi^2)*n*\ln(n) + O(n)$$ The proof for the geometric mean goes ...
David desJardins's user avatar
12 votes

Asymptotic behavior of a certain oscillatory integral

$$I(x):=\int_{0}^{\infty}\frac{e^{i r}}{r^{\frac{1}{2}}}\int_{0}^{\infty}\frac{e^{-s}}{s^{\frac{1}{2}}}\frac{r}{sx+\sqrt{sxr}+r}ds dr.$$ To aid the asymptotic analysis, I regularize $I(x)$ by ...
Carlo Beenakker's user avatar
12 votes
Accepted

Lindelöf hypotheses for derivatives of zeta

The Lindelöf hypothesis yields the same bound for each derivative of $\zeta(s)$ via Cauchy's formula. Indeed, let $n\in\mathbb{N}$, $\sigma\in\mathbb{R}$, $T\in(1,\infty)$, $\varepsilon\in(0,1/2)$. ...
GH from MO's user avatar
  • 99.1k
11 votes
Accepted

On an asymptotic integral decay

I'll change $[-1,0]$ to $[0,1]$ (so $t\to -t$), which seems easier on the brain. $f(t)=e^{-1/t}$ is a counterexample: For $0<t<1/\sqrt{\lambda}$, we have $f\le e^{-\sqrt{\lambda}}$, so this part ...
Christian Remling's user avatar
11 votes

Is there an upper bound on the number of representations as a sum of squares?

It is known since Gauss that $$r_3(n)=12\dfrac{h(D)}{w(D)/2}(1-(D/2))\sum_{d\mid f}\mu(d)(D/d)\sigma(f/d)\;,$$ where $-n=D(2^vf)^2$, $D$ a fundamental discriminant, $v\ge-1$, $h(D)$ is the class ...
Henri Cohen's user avatar
  • 11.7k
10 votes
Accepted

Asymptotics of an integral requested

Change variables to $y=n(1-x)$, which gives $$ I_n=\int\limits_{0}^{n}\left( 1-\frac{y}{n}\right)^{n-1} \sqrt{1-\frac{\log y}{\log n}}\ {\rm d}y\ , $$ then dominated convergence should allow passing ...
Abdelmalek Abdesselam's user avatar
9 votes
Accepted

Does the Riemann hypothesis predict a bound for this prime-counting function?

The Riemann hypothesis is equivalent to the following statement: $$f(x)=\mathrm{li(x)}-\frac{x}{\log x}+O(\sqrt{x}),\qquad x\geq 2.$$ Note that $$\mathrm{li(x)}=\mathrm{li(2)}+\frac{x}{\log x}-\frac{2}...
GH from MO's user avatar
  • 99.1k
9 votes
Accepted

On a paper by Dimitrie Pompéiu and on one (in two parts) by Edmund Landau

Below are both references: links then screenshots. The first reference can be found in full here. In case permissions etcetera should change, below are screenshots of the pages: The second reference ...
Benjamin Dickman's user avatar
9 votes

Asymptotics of a strange oscillatory function

Yes, to (1) and (2), but the argument below is much too crude to identify $c$. The obvious attempt is to use $\sin t\ge 2t/\pi$, $0\le t\le \pi/2$ (or something similar), for the terms with large $n$. ...
Christian Remling's user avatar
8 votes
Accepted

Asymptotic behavior of a certain oscillatory integral

We can evaluate $I(x)$ explicitly, and then asymptotically. Indeed, using the substitution $s=ru/x$, we get \begin{equation*} I(x)=\frac1{\sqrt x}\lim_{R\to\infty}J_R(x), \tag{1}\label{1} \end{...
Iosif Pinelis's user avatar
8 votes
Accepted

Series with the smallest number whose square is divisible by $n$

I couldn't find a reference, but (as noted in the OEIS page) if we have $k = a b^2$ with squarefree $a$ then $a(k) = ab$, so $$\begin{align*} \sum_{k\leq x}\frac1{a(k)} &= \sum_{a b^2 \leq x} \...
Command Master's user avatar
7 votes

Probability of large gcd

To get a bound which is worse than the one of GH from MO asymptotically, but which doesn't require any case checking, we can do the following: if $\gcd(t, N) = k$, then $\frac{N}{k} = d$ which is an ...
Aleksei Kulikov's user avatar
7 votes
Accepted

Probability of large gcd

The statement is true in the stronger form that $$\Pr[\gcd(t, N)>N^{3/4}] < N^{-1/2}.$$ Indeed, the probability that $\gcd(t,N)$ equals a given $k\mid N$ is at most $1/k$. For $k>N^{3/4}$, ...
GH from MO's user avatar
  • 99.1k
7 votes
Accepted

Randomly removing length 1 intervals in an interval (a fragmentation process)

This is exactly the so-called "Renyi parking process." Renyi proved $$\varphi(x) = cx + c - 1 + o(1)$$ where $$c = \int_0^\infty \exp\left( -2 \int_0^x \frac{1 - e^{-y}}{y}\,dy \right)\,dx \...
Marcus M's user avatar
  • 909
6 votes
Accepted

Binomial coefficient asymptotics

Yes, this is a special case of the de Moivre–Laplace theorem.
Iosif Pinelis's user avatar
5 votes
Accepted

Asymptotic behavior of the integral that contains $\delta$ function

$$P(s)=2\int_{-\infty}^{\infty}{\rm d}p\int_{-\infty}^{\infty}{\rm d}q\ \delta\left(\frac{p^2+(p^2-q^2)^2}{p^2+4p^4}-s\right)e^{-\left(p^2+q^2\right)/a^2}$$ $$\qquad=2\int_{0}^{\infty}\frac{{\rm d}x}{\...
Carlo Beenakker's user avatar
5 votes
Accepted

On the asymptotic behaviour of the series $\sum_{n=1}^{\infty} \left ( \frac{\zeta(ns)}{n}+\frac{s}{1-ns}\right )$ near $s=0$

Proposition. The following approximation holds true: $$f(s)=\frac{1}{2}\log s+O(1),\qquad s\in(0,1).$$ Proof. Following Carlo Benakker, we remark that $$\zeta(s)+\frac{s}{1-s}=-\frac{1}{2}+O(s),\qquad ...
GH from MO's user avatar
  • 99.1k
4 votes
Accepted

The asymptotic behavior of $F(\lambda):=\sum_{k=0}^{\infty}\frac{\Gamma{(a k)}}{\Gamma{(b k)}}{\lambda}^{-k}$, $b>a>0$

$\newcommand{\Ga}{\Gamma}$The value of \begin{equation*} r_k:=\frac{\Ga(ak)}{\Ga(bk)} \end{equation*} is undefined at $k=0$. So, let \begin{equation*} r_0:=\lim_{k\to0}\frac{\Ga(ak)}{\Ga(bk)}=\...
Iosif Pinelis's user avatar
4 votes

Slick proof of Stirling's Formula?

I've played around with this a bit. I have a slick lower bound, but not a slick upper bound. We start with the $\Gamma$-integral: $$n! = \int_{x=0}^\infty x^n e^{-x} dx = \int_{y=-n}^\infty (n+y)^n e^{...
David E Speyer's user avatar
4 votes
Accepted

A limit arising from Mellin Inversion: How to compute a specific term of an asymptotic series?

As $s\to0^+$, we have $s\Gamma(s)=\Gamma(s+1)\to1$, $x^s\to1$ unless $x=0$, and $e^{-s f(n)}\to1$ for each $n$. So, by (say) the Fatou lemma, $$ L:=\lim_{s\to0^+} \left[s \sum_{n=0}^{\infty} e^{-s f(...
Iosif Pinelis's user avatar
4 votes
Accepted

How fast does this summation grow?

Let us obtain the asymptotic of $\log S_n$, where $\log$ denotes the base-$2$ logarithm, \begin{equation*} S_n:=\sum_{k=1}^n\prod_{l=1}^k\binom{2^n}{2^l}^i =\sum_{k=1}^n a_{n,k}, \end{...
Iosif Pinelis's user avatar
4 votes
Accepted

$L^1$ error between indicator function and smoothed out version

Yes, this works, and the only ingredient we need is the estimate $\int_r^{\infty} e^{-t^2}\, dt\lesssim e^{-r^2}$. We then have (for example) \begin{align*} \int_r^{\infty} |f_r(x)|\, dx &=\frac{1}...
Christian Remling's user avatar
4 votes
Accepted

Asymptotic solution of a system of ODEs

This is my comment as an answer. Using Mathematica's ParametricNDSolve[] with 50 digits precision as well as a series expansion around $u=\infty$, I get \begin{align} a(u) &= -1.02418609585 u^{-1} ...
Fred Hucht's user avatar
  • 2,705
4 votes

On the asymptotic behaviour of the series $\sum_{n=1}^{\infty} \left ( \frac{\zeta(ns)}{n}+\frac{s}{1-ns}\right )$ near $s=0$

We seek the small-$s$ asymptotics of the sum $$f(s)=s\sum_{n=1}^\infty g(ns),\;\;g(x)=\frac{\zeta(x)}{x}+\frac{1}{1-x}.$$ First note that $$\lim_{s\rightarrow 0}\sum_{n=\lfloor{2/s}\rfloor+1}^\infty ...
Carlo Beenakker's user avatar
3 votes

Asymptotic scaling of mean and variance for non-central chi distribution

For simplicity, I take equal $\mu_i=\mu$, $\sigma_i^2=\sigma$. The generalisation to arbitrary $\mu_i,\sigma_i$ is straightforward, $k(\mu/\sigma)^2\mapsto\sum_{i=1}^k(\mu_i/\sigma_i)^2={\cal O}(k).$ ...
Carlo Beenakker's user avatar
3 votes
Accepted

Extreme case bounds on Diophantine approximation

If $\alpha$ is a real irrational number, then there are infinitely many coprime integers $p,q$ with $q > 0$ such that $$\displaystyle \left \lvert \alpha - \frac{p}{q} \right \rvert < \frac{1}{q^...
Stanley Yao Xiao's user avatar
3 votes
Accepted

Estimating the bound of the integral over whole $\mathbb{R}$ of the Taylor remainder term?

$\newcommand\R{\mathbb R}$We want to bound the ratio \begin{equation*} R:=\frac ND, \end{equation*} where \begin{equation*} N:=\iint_{\R^2}(F(x,w)-F(y,w))^2 \mu(dx)\mu(dy),\quad D:=\...
Iosif Pinelis's user avatar
3 votes

How fast does this summation grow?

For $$A_n=\sum_{k=1}^n\prod_{l=1}^k\binom{2^n}{2^l}$$ it seems that $$\log(\log(A_n))\sim \alpha +n \,\log(2) \qquad \qquad \alpha \sim -0.383$$
Claude Leibovici's user avatar

Only top scored, non community-wiki answers of a minimum length are eligible